three items that have a capacity greater than 10 liters are Large cooking pots, Backpacks, Water coolers.
Here are three items that have a capacity greater than 10 liters:
Large cooking pots - Many cooking pots used in commercial kitchens have a capacity of 10 liters or more.
Backpacks - There are many backpacks available with a capacity greater than 10 liters, including hiking backpacks, travel backpacks, and backpacks designed for carrying large amounts of equipment or gear.
Water coolers - Many water coolers have a capacity of 10 liters or more, making them useful for providing drinking water for a large group of people over an extended period of time.
To know more about capacity, visit: brainly.com/question/23944576
#SPJ4
According to the empirical rule, the bell or mound shaped distribution will have approximately 68% of the data within what number of standard deviations of the mean
The correct option is option a) One standard deviation.
According to the empirical rule, the bell or mound shaped distribution will have approximately 68% of the data within one standard deviations of the mean.
According to the empirical rule, the bell or mound-shaped distribution will have approximately 68% of the data within one standard deviation of the mean. This means that if the data is normally distributed, then about 68% of the data points will fall within one standard deviation above or below the mean.
Similarly, the empirical rule states that approximately 95% of the data will fall within two standard deviations of the mean, and about 99.7% of the data will fall within three standard deviations of the mean.
This means that if the data is normally distributed, then 95% of the data points will fall within two standard deviations above or below the mean, and 99.7% of the data points will fall within three standard deviations above or below the mean.
It is important to note that the empirical rule is based on the assumption that the data is normally distributed. If the data does not follow a normal distribution, then the empirical rule may not apply.
Therefore, the answer to the question is (a) One standard deviation, (b) Two standard deviations, and (c) Three standard deviations. Option (d) Four standard deviations and (e) Four standard deviations are not correct, and option (f) None of the above is partially correct as it excludes options (a), (b), and (c), but option (g) All of the above is not correct as options (d) and (e) are incorrect.
To learn more about standard deviations, refer here:
https://brainly.com/question/23907081#
#SPJ11
Diane is 25 years older than her son Craig. The sum of their ages is 105. How old are Diane and Craig?
Answer:
Let Craigs age = x
Therefore Dianes age = 25 + x ( since she's 25 years older)
Dianes + Craigs age = (25 + x) + x = 105 (They said both of their ages sums up to 105)
25 + 2x = 105
2x = 80
x = 40
Craigs age is 40 years old.
Dianes age is 65 years old.
Answer:
40 and 65
Step-by-step explanation:
Craig is 40.
Diane is 65.
40+25=65.
Diane is 25 years older than Craig.
40+20=105
Use the following statements to write a compound
statement for the disjunction -p or -q. Then find its truth
value.
p: There are 14 inches in 1 foot.
q: There are 3 feet in 1 yard.
The disjunction of -p or -q can be written as (-p) v (-q). So, we have to find the truth value of (-p) v (-q). So, the compound statement for the disjunction of -p or -q is (-p) v (-q), and its truth value is true.
using the following statements: p: There are 14 inches in 1 foot.
q: There are 3 feet in 1 yard.
Solution: We know that 1 foot = 12 inches, which means that there are 14 inches in 1 foot can be written as 14 < 12. But this statement is false because 14 is not less than 12. Therefore, the negation of this statement is true, which gives us (-p) as true.
Now, we know that 1 yard = 3 feet, which means that there are 3 feet in 1 yard can be written as 3 > 1. This statement is true because 3 is greater than 1. Therefore, the negation of this statement is false, which gives us (-q) as false.
Now, we can use the values of (-p) and (-q) to find the truth value of (-p) v (-q) using the disjunction rule. The truth value of (-p) v (-q) is true if either (-p) or (-q) is true or both (-p) and (-q) are true. Since (-p) is true and (-q) is false, the disjunction of (-p) v (-q) is true. Hence, the compound statement for the disjunction of -p or -q is (-p) v (-q), and its truth value is true.
For more questions on: compound statement
https://brainly.com/question/28794655
#SPJ8
A small business buys a computer for $3000. After 4 years the value of the computer is expected to be $200. For accounting purposes the business uses straight-line depreciation (this means that if V is the value of the computer at time t, then a linear equation is used to relate V and t) to assess the value of the computer at a given time.
(a) Find a linear equation that models the value V of the computer t years since its purchase.
V(t)=−700t+300
The linear equation that models the value V of the computer t years since its purchase is V(t) = -700t + 3000
How to find linear equationT do this, use the two given data points to write a linear equation in slope-intercept form
Thus,
V(t) = mt + b
where
m is the slope and
b is the y-intercept.
At t=0 (the time of purchase), the value of the computer is $3000
V(0) = m(0) + b
3000 = b
After 4 years, the value of the computer is $200
V(4) = m(4) + b
200 = 4m + 3000
Collect like terms
200-3000 =4m
Divide both side by 4
m=-2800/4
m = -700
b = 3000
Therefore, the linear equation that models the value V of the computer t years since its purchase is V(t) = -700t + 3000
Learn more on linear equation on https://brainly.com/question/2030026
#SPJ4
Ashley had 4/ 5 of a spool of yarn. She used 2/5 of it for her project. What fraction of the spool was used for her project? Write your answer in simplest form
Ashley used 8/25 of the spool for her project.
To determine the fraction of the spool that Ashley used for her project, we need to multiply the fraction of the spool she had (4/5) by the fraction she used (2/5):
(4/5) * (2/5) = 8/25
Know more about fraction here:
https://brainly.com/question/10354322
#SPJ11
how many eight digit numbers contain the digit 2 once, the digit 3 twice, the digit 4 thrice, and more 5s than threes? leading 0s are allowed.
0.0405 is the number that contain the digit 2 once, the digit 3 twice, the digit 4 thrice, and more 5s than threes, leading 0s are allowed.
According to collect from the data
Let us assume that the data as followed by
How many eight digit numbers contain the digit 2 exactly once, the digit 3 exactly twice, the digit 4 exactly thrice and more 5s than threes.
Let many eight digit numbers contain the digit 2 once, the digit 3 twice the digit 4 thrice and more 5s than threes.
2,3,3,4,4,4
now then:
= 8!/2!3!3!4!4!4!
= 8.7.6.5.4!/2×6×6×24×24×4!
= 35/864
Now 0.405 then the eight digit number contains the digit 2 once the digit 3 .
Learn more about Permutations and combinations here:
brainly.com/question/4658834
#SPJ4
What is the missing statement for step 7?
there are 18 major sea islands in the queen elizabeth islands of canada. there are 15 major lakes in saskatchewan, canada. (a) if you are planning a trip to visit one of these islands, followed by one of these lakes, how many different trips could you make? (b) if you plan to visit either one of these lakes or one of these islands, how many different visits could you make?
(a) Total number of trips = 270
(b) There are 33 different visits that could make.
We have the information:
There are 18 major sea islands in the queen Elizabeth islands of Canada. There are 15 major lakes in Saskatchewan, Canada.
Let us say that A indicates the major sea Islands and B indicates the major lakes.
Taking both the cases as events, we will use multiplication principle.
Based on the information, the event A occurs in m ways and the event B will occur in n ways. So, the two events will occur in m×n ways.
(a) A × B = 18 × 15
Total number of trips = 270
(b) |A| = 18
|B| = 15
A ∩B = θ (as we cannot visit both an island and lake)
Addition Principle: There are |A| + |B| ways to choose an element form
A ∪ B , when A and B are the finite sets with A ∩B = θ
|A ∪ B| = |A| + |B| = 18 + 15 = 33
Thus, There are 33 different visits that could make.
Learn more about Addition principle at:
https://brainly.com/question/12904549
#SPJ4
What is the perimeter of 42 feet long and 18 feet wide
Answer:
Step-by-step explanation:120
The perimeter is :
↬ A = 576 ft^2Work:
Assuming you're asked to find the perimeter of a rectangle, I set to work.
The formula is A = LW, where A is the area, L is the length and W is the width.
Diagram
\(\boxed{\begin{tabular}{c}~~~~~~~~~~~~~~~~~~~~~~~~~~~~~~~~~~~~~~~\\\\\\\\\end{tabular}}}\sf{~~42\:ft}\\\sf{~~~~~~18~ft}\)
Plug in the values.
\(\boldsymbol{A=LW}\)\(\boldsymbol{A=756~ft^2}\)Hence, A = 756 ft².NEED ANSWER ASAP!!! WILL GIVE BRAINLIEST TO WHO ANSWERS FIRST!
Solve for x: 2x − 2 > 4x + 6. (5 points)
x −4
x 4
Answer:
x < - 4
Step-by-step explanation:
Given
2x - 2 > 4x + 6 ( subtract 2x from both sides )
- 2 > 2x + 6 ( subtract 6 from both sides )
- 8 > 2x ( divide both sides by 2 )
- 4 > x , thus
x < - 4
I NEED HELP !!!!!!!!!!!
Answer:
Step-by-step explanation:
2x - 3 < x + 4
x - 3 < 4
x < 7
Open dot at seven with a line to the left.
7 + x > 5
x > -2
Open dot at negative 2 with a line to the right.
Find the area of an equilateral triangle (regular 3-gon) with 6-inch sides. Round your answer to the nearest hundredth.
James is solving a number puzzple that involves three integers, a,b, and c, where, c is a positive integer.The product of a and b is 6.The product of a and c is -4.The product of b and cis -6.
Answer:
a = -2
b = -3
c = 2
Step-by-step explanation:
a*b = 6 {The product of a and b is 6]
a*c = -4 [The product of a and c is -4]
b*c = -6 [The product of b and c is -6]
- Rearrange:
1) a*c = -4 to a = (-4/c)
2) b*c = -6 to b = (-6/c)
Now substitute:
a*b = 6
(-4/c)*(-6/c) = 6
(24/c^2) = 6
c^2 = (24/6)
c^2 = 4
c = 2
use this value of c to find a and b:
a = (-4/c)
a = (-4/2)
a = -2
b = (-6/c)
b = (-6/2)
b = -3
Trigonometry Question:
Find the length of side AB
Give your answer to 1 decimal place
Answer:
5.6 cm
Step-by-step explanation:
use the cosine ratio to get the answer since AB is the adjacent side and the hypotenuse has already been given
Lucas bought a dress shirt, a hat, and a belt. The price of the dress shirt is $3 more than the price of the belt.
The hat costs $5 less than the dress shirt. He paid a total of $49. What is the price of each item?
Answer:
Dress shirt: 19.3333333
hat: 14.3333333
belt: 15.3333334
Step-by-step explanation:
49÷3= 16.3333333
16.3333333+3= 19.3333333 dress shirt.
19.3333333-5= 14.3333333 hat.
19.3333333+14.3333333= 33.6666666
49-33.6666666= 15.3333334 belt.
9,892,846,437=9,000,000,000+[answer]+90,000,000+2,000,000+800,000+40,000+6,000+[answer]+30+7
Answer:
it all correct because it is expanding
Policies Current Attempt in Progress On May 1, 2021, Sheffield Company sells office furniture for $300000 cash. The office furniture originally cost $746800 when purchased on January 1, 2014. Depreciation is recorded by the straight-line method over 10 years with a salvage value of $80200. What gain should be recognized on the sale? (Hint: Use 7.333333 for years used in calculation.) O $44540. O $22220. O $84080. O $42040. Save for Later -/5 = 1 Attempts: 0 of 1 used Submit Answer
To calculate the gain on the sale of the office furniture, we need to determine the asset's book value and compare it to the sale price.
First, let's calculate the accumulated depreciation on the furniture. The furniture was purchased on January 1, 2014, and the straight-line depreciation method is used over 10 years with a salvage value of $80,200.
Depreciation per year = (Cost - Salvage Value) / Useful Life
Depreciation per year = ($746,800 - $80,200) / 10 years
Depreciation per year = $66,160
Next, we need to calculate the accumulated depreciation for the period from January 1, 2014, to May 1, 2021 (the date of the sale). This is approximately 7.33 years.
Accumulated Depreciation = Depreciation per year × Years
Accumulated Depreciation = $66,160 × 7.33 years
Accumulated Depreciation = $484,444.80
Now, we can calculate the book value of the furniture:
Book Value = Cost - Accumulated Depreciation
Book Value = $746,800 - $484,444.80
Book Value = $262,355.20
Finally, we can calculate the gain on the sale:
Gain on Sale = Sale Price - Book Value
Gain on Sale = $300,000 - $262,355.20
Gain on Sale = $37,644.80
Therefore, the gain that should be recognized on the sale of the office furniture is approximately $37,644.80.
To know more about depreciation, refer here:
https://brainly.com/question/30531944
#SPJ11
The gain that should be recognized on the sale of the office furniture is $84,080.
The gain is calculated by subtracting the equipment's book value from the sale price. This gain will be reported on the company's income statement. Here is how to calculate the gain:First, find the equipment's book value using the straight-line method of depreciation.
Straight-line depreciation is calculated by taking the difference between the equipment's original cost and its salvage value, and then dividing it by the number of years the equipment is used. The annual depreciation expense is then multiplied by the number of years the equipment is used to find the equipment's book value at the end of its useful life.
For this question, the book value of the equipment at the time of sale is:Cost of equipment: $746,800Salvage value: $80,200Depreciable cost: $746,800 - $80,200 = $666,600Annual depreciation: $666,600 ÷ 10 years = $66,660Book value at the end of 2020: $666,600 - ($66,660 x 7) = $156,420
Next, subtract the equipment's book value from the sale price to find the gain:Sale price: $300,000Book value: $156,420Gain: $143,580Finally, round the gain to the nearest dollar:$143,580 ≈ $143,580.00So the gain that should be recognized on the sale of the office furniture is $84,080.
learn more about gain here:
https://brainly.com/question/31218742
#SPJ11
Question is in image
The radius of the spherical part is 3 inches and the length of the tube is 21 cm.
What is the volume of a cylinder?The capacity of a cylinder, which determines how much material it can hold, is determined by the cylinder's volume. There is a formula for the volume of a cylinder that is used in geometry to determine how much of any quantity, whether liquid or solid, can be immersed in it uniformly.
Given that the volume when the water is fully dipped is 4554 / 7 cm³. The volume when the length is 9 cm empty is 396 cm³.
The two equations can be formed as below:-
4554 / 7 = πr²l + (2/3)πr³
396 = πr²( l - 9 ) + (2/3)πr³
Subtract the second equation from the first,
( 4554 / 7 ) - 396 = 9πr²
9πr² = 254.5
r² = 9
r = 3 cm
The length will be calculated as:-
4554 / 7 = πr²l + (2/3)πr³
4554 / 7 = π( 3 )²l + (2/3)π(3)³
l = 21 cm
Therefore, the tube's length is 21 cm, and the spherical part's radius is 3 inches.
To know more about volume follow
https://brainly.com/question/22716418
#SPJ1
BRAINLIEST!!!!!!!!!!!!!!!!!!
Answer:
41 yw
Step-by-step explanation:
Which of the following parts does not belong to a theoretical:
A. The hypotheses corresponding to the model
B. The operationalization of the used constructs of the model
C. A logical explanation of the relationship within a model
D. A graphical respresentation of the model
The operationalization of the used constructs of the model is not typically considered a part of the theoretical aspects of a model. The correct option is (B).
Operationalization refers to the process of defining and measuring the variables or constructs in a study. It involves turning abstract concepts into observable and measurable variables.
On the other hand, the theoretical parts of a model typically include:
A. The hypotheses corresponding to the model: This refers to the specific statements or predictions derived from the theoretical framework of the model.
Hypotheses provide testable expectations about the relationships between variables in the model.
C. A logical explanation of the relationship within a model: This involves providing a theoretical rationale or logical explanation for the expected relationships between variables in the model.
It is the theoretical framework that underlies the model and helps in understanding the underlying mechanisms or processes.
D. A graphical representation of the model: Graphical representation, such as diagrams or visual models, can be used to depict the structure or relationships within a model. It helps in visualizing the connections between variables and understanding the overall framework.
Therefore, B. The operationalization of the used constructs of the model does not belong to the theoretical parts of a model.
To know more about operationalization refer here:
https://brainly.com/question/30636311#
#SPJ11
Which expressions represent “the sum of 3 and n”?
Select all that apply
Answer:
Step-by-step explanation:
a- 3+n
Answer:
First option and Second option
Step-by-step explanation:
Hope it helps. Happy Easter Tuesday. Have a great day.
a dataset on 91 roller coasters lists the duration of the ride in seconds in addition to the drop height in feet for some of the coasters. one coaster, the tower of terror, is unusual for having a large drop but a short ride. after setting it aside, a regression to predict duration from drop for the remaining 90 coasters has r29.4%. complete parts a through c.a) What are the variable and units in this regression? The predictor variable is ___ in units of ___ and the response variable is ___ in units of ___ b) What units does the slope have? Feet, Seconds per foot, Seconds, Feet per second. c) Is the slope probably positive or probably negative? Explain. The slope is probably ____ because ___
(a) The predictor variable is Fall in feet and the response variable is Duration in seconds.
(b) The slope is in seconds per foot.
(c) The slope may be negative, as greater drop heights generally result in longer travel times.
(a) The predictor variable in this regression is Drop, which represents the drop height of the roller coaster, in feet, in feet. The response variable is Duration, which represents the duration of the trip, in seconds, in seconds.
(b) The units of slope in this regression are seconds per foot. This means that for each unit of increase in descent height in feet, the travel time will increase or decrease the value of the slope, measured in seconds per foot.
(c) The slope can be negative, as greater drop heights generally result in longer ride times, and the Tower of Terror roller coaster, which was exceptionally short despite its large drop height, has were removed from the analysis.
Therefore, the remaining 90 roller coasters in the dataset likely exhibit a negative relationship between drop height and time, meaning that as drop height increases, ride time decreases .
The Low R-value squared 29.4% means that the relationship between roller coaster drop and ride time in the data set is highly variable, but the negative slope indicates a general downward trend.
Learn more about Slope:
https://brainly.com/question/3605446
#SPJ4
Katherine and her children went into a bakery and where they sell cupcakes for $2.50 each and donuts for $1.25 each. Katherine has $25 to spend and must buy at least 14 cupcakes and donuts altogether. If xx represents the number of cupcakes purchased and yy represents the number of donuts purchased, write and solve a system of inequalities graphically and determine one possible solution.
Answer:the ration to x and b that equals c is about 3 dollars
Step-by-step explanation:
Write an equation in slope-intercept form for the line with slope -2/3 and y-intercept -3
what is y=6x^2 +96x +388 in vertex form?
please show work
Answer:
y = 6(x + 8)² + 4
Step-by-step explanation:
The equation of a parabola in vertex form is
y = a(x - h)² + k
where (h, k) are the coordinates of the vertex and a is a multiplier
Using the method of completing the square.
y = 6x² + 96x + 388
The coefficient of the x² term must be 1, so
factor out 6 from 6x² + 96x
y = 6(x² + 16x ) + 388
To complete the square
add/subtract (half the coefficient of the x- term)² to x² + 16x
y = 6(x² + 2(8)x + 64 - 64) + 388
= 6(x + 8)² - 384 + 388
y = 6(x + 8)² + 4 ← in vertex form
Help!! Will give the almighty great crown
Answer:
G and D
Step-by-step explanation:
look closelyfind the coordinate linesgo seedrag each tile to the correct box. a company employs 650 people and wishes to survey a sample of its employees about the company culture. to avoid bias in the survey, the human resources director creates a list of all the employees and randomly selects 150 of them to complete the survey. which description matches each term? the 650 employees in the
To avoid bias in the survey, the human resources director creates a list of all the employees and randomly selects 150 of them to complete the survey.
In this scenario, we have a company that employs 650 people and wishes to survey a sample of its employees about the company culture. Let's match each term with its corresponding description:
1. Population: The population refers to the entire group of individuals that the survey aims to represent. In this case, the population is the total number of employees in the company, which is 650.
2. Sample: A sample is a subset of the population that is selected for data collection and analysis. It represents a smaller portion of the population. In this scenario, the sample consists of the 150 employees randomly selected by the human resources director.
3. Random Selection: Random selection is the process of choosing individuals from the population in a way that ensures each member has an equal chance of being included in the sample. By randomly selecting the 150 employees, the human resources director avoids bias and increases the likelihood that the sample represents the entire population.
4. Survey: A survey is a data collection method used to gather information from individuals within the sample. In this case, the selected employees will be asked to complete a survey about the company culture.
By randomly selecting 150 employees from the total population of 650, the company aims to create a sample that is representative of the entire workforce. This helps to avoid bias and increase the generalizability of the survey findings. The survey responses from the selected employees will provide insights into the company culture, which can then be used to make informed decisions or improvements. It's important to note that the quality of the survey and the representativeness of the sample can impact the validity and reliability of the survey results. Therefore, careful consideration should be given to the sampling method and survey design to ensure accurate and meaningful findings.
Learn more about randomness here:
brainly.com/question/29315928
#SPJ11
(a) Attendance at the Accra Sports Stadium was alysed by the General Secretary, Prosper Harrison Addo. The analysis demonstrated that spectators consisted of 70% males. If seven people are randomly selected from the spectators during a football match, What is the probability that 4 of them are males? (3 marks) i 11. Find the probability that at most 5 of them are females (4 marks)
a) The probability of randomly selecting 4 males out of 7 spectators, given that 70% of the spectators are males, can be calculated using the binomial probability formula.
b) To find the probability that at most 5 of the randomly selected spectators are females, we need to calculate the cumulative probability of selecting 0, 1, 2, 3, 4, and 5 females from the total number of selected spectators.
a) To calculate the probability of selecting 4 males out of 7 spectators, we can use the binomial probability formula:
P(X = k) = C(n, k) * p^k * (1 - p)^(n - k)
Where:
- n is the total number of trials (number of people selected)
- k is the number of successful trials (number of males selected)
- p is the probability of success in a single trial (probability of selecting a male)
- C(n, k) is the binomial coefficient, calculated as C(n, k) = n! / (k! * (n - k)!)
In this case, n = 7, k = 4, and p = 0.70 (probability of selecting a male). Therefore, the probability of selecting 4 males out of 7 spectators is:
P(X = 4) = C(7, 4) * (0.70)^4 * (1 - 0.70)^(7 - 4)
b) To find the probability that at most 5 of the selected spectators are females, we need to calculate the cumulative probability of selecting 0, 1, 2, 3, 4, and 5 females. This can be done by summing the individual probabilities for each case.
P(X ≤ 5 females) = P(X = 0) + P(X = 1) + P(X = 2) + P(X = 3) + P(X = 4) + P(X = 5)
To calculate each individual probability, we use the same binomial probability formula as in part a), with p = 0.30 (probability of selecting a female).
Finally, we sum up the probabilities for each case to find the probability that at most 5 of the selected spectators are females.
To learn more about probability Click Here: brainly.com/question/31828911
#SPJ11
Factor each completely.
v2-4v-12
Answer:
I think the answer is (v+2) x (v-6)
evaluate the triple integral f(x,y,z) = x^2 y^2 over the region p<2
The triple integral is equal to ∫∫∫ f(x, y, z) dV using spherical coordinates is equal to 64π/21 .
Use spherical coordinates to evaluate this triple integral over the given region.
The region p < 2 is a sphere centered at the origin with radius 2.
In spherical coordinates, this region can be described by,
0 ≤ ρ ≤ 2
0 ≤ θ ≤ 2π
0 ≤ φ ≤ π
The volume element in spherical coordinates is ρ² sin φ dρ dφ dθ.
The triple integral can be written as,
∫∫∫ f(x, y, z) dV
= \(\int_{0}^{2}\int_{0}^{\pi}\int_{0}^{2\pi }\) (ρ² sin φ)(ρ⁴ sin²φ cos²θ sin²θ) dρ dφ dθ
= \(\int_{0}^{2}\int_{0}^{\pi}\int_{0}^{2\pi }\) (ρ⁶ sin³φ cos²θ sin⁵ θ) dρ dφ dθ
= \(\int_{0}^{2}\)(ρ⁶/7) \(\int_{0}^{\pi }\) (sin³ φ) \(\int_{0}^{2\pi }\) (cos² θ sin⁵θ) dθ dφ dρ
The innermost integral evaluates to π/8.
The second integral can be evaluated using the substitution u = cos φ, du = -sin φ dφ, which gives,
\(\int_{0}^{\pi }\)(sin³ φ) dφ
= -\(\int_{1}^{-1}\)(1-u²) du
= 4/3
The outer integral evaluates to (2⁷)/7.
Triple integral is equal to
∫∫∫ f(x, y, z) dV
= (2⁷/7) (4/3) (π/8)
= (32/7)π/6
= 64π/21
Therefore, the triple integral is equal to ∫∫∫ f(x, y, z) dV = 64π/21 .
learn more about triple integral here
brainly.com/question/31383102
#SPJ4